Find the image of (-1, 2) reflected across the y-axis.

Answers

Answer 1

Answer: (1,2)

Step-by-step explanation:

Reflecting across the y-axis means [tex](x,y) \longrightarrow (-x,y)[/tex]

So, [tex](-1,2) \longrightarrow \boxed{(1,2)}[/tex]


Related Questions

Write as an equation: The product of a number and 12 is 78.
A. 12n=78
12
n
=
78
B. n+12=78
n
+
12
=
78
C. 12−n=78
12

n
=
78
D. 12=78n

Answers

Answer: A) 12n = 78

Detailed Solution:

Let ‘n’ be the Unknown Number.

Product of ‘n’ and 12 = 12 * n = 12n

Therefore,
12 * n = 78
=> 12n = 78
The answer is 12n = 78. Why? Well, the product of a number and 12 can be written as 12n because it is a coefficient which includes a variable for an unknown value. Since there is an unknown value (that’s called “a number” in this expression), 12n is the standard form of this representation. Since the product of a number and 12 is 78, we can use an equal sign followed by a 78 to show that 12n is equal to 78. If you need to better understand this, let me know and I will gladly assist you.

Anthony is planning a trip using a map with the scale of 1 cm to 15 miles it’s a destinations 15 cm away on the map how far away in Miles as the destination

Answers

The distance of Miles' destination is 225 miles.

What is the distance of the destination?

A scale drawing is a reduced form in terms of dimensions of an original image / building / object. The map is a scale drawing of a larger town. The scale drawing is reduced by constant dimensions.

Actual distance = (15 x 15) / 1 = 225 miles

To learn more about scale drawings, please check: https://brainly.com/question/26388230

#SPJ1

Find the radius of the given circle with the given central angle and arc length. Round your answer to the nearest tenth.
Show work for full credit.
130*
19.6 cm

Answers

Answer: 8.6

Step-by-step explanation:

[tex]19.6=2\pi r\left(\frac{130}{360} \right)\\\\r=\frac{19.6}{2\pi \left(\frac{130}{360} \right)}\\\\r \approx 8.6[/tex]

32. Find the ratio of the side lengths of the small triangle to the large
triangle.

3/10

6/20

3/2

2/3

Answers

Answer:

2/3

Step-by-step explanation:

that the ratio of that number

Look At The Graph. How many nails are less than 1 1/2 inches long?

A)7
B)17
C)6
D)15

Answers

Answer:

D 15

Step-by-step explanation:

Count all of the x's less than 1 1/2  =  15

A glass contains alcohol and water in the ratio 1:4. A second glass contains the same quantity of liquid, but this time the ratio of alcohol to water is 2:3. Each glass is emptied into a third glass. What is the ratio of alcohol to water in the final mixture?
A glass contains alcohol and water in the ratio 1:4. A second glass contains the same quantity of liquid, but this time the ratio of alcohol to water is 2:3. Each glass is emptied into a third glass. What is the ratio of alcohol to water in the final mixture?

Answers

Answer:

3:7

Step-by-step explanation:

So to solve this problem, you have to understand what the ratio 1:4 and 2:3 means. The 1:4 ratio in the first equation means that for "each unit of alcohol" there is 4 of those units of water. So let's say I had 2 gallons of alcohol and mixed it with 8 gallons of water. This means for each gallon of alcohol, there is 4 gallons of water, or in other words a 1:4 ratio. This can be described as a percentage as well. For each 5 gallons there are 4 gallons of water, and 1 gallon of alcohol or 20% is alcohol. So let's just say that x=alcohol and y=water, this means that: [tex]x+y=c[/tex]  where c is the total amount in the glass. This means that: [tex]x=0.20c[/tex]

Let's do the same thing to the second equation. the ratio means that for every 2 units of alcohol there are 3 units of water. This means for every 5 gallons of the mixture there is 2 units of alcohol which is 40%. In this case let's also say that j=alcohol and k=water. This means that: [tex]j+k=c[/tex] and that: [tex]j=0.40c[/tex].

So if we're going to add the two glasses, we simply add the two sides, and get: [tex]j+x+k+y=2c[/tex]. Now remember how can can express j and x in terms of c, since it's a certain percentage of c (the entire thing). This means that we get: [tex](0.4c+0.2c)+x+k=2c[/tex] Now we can add like terms to get the equation: [tex]0.6c+y+k=2c[/tex]. We can find how much 0.6c is to 2c by dividing the 2, in doing so we get that 0.6c/2c = 0.3, or in other words the 0.6c is only 30% of the final mixture, and since the 0.6c represents the alcohol in this mixture, that means that's the percentage of alcohol. To write this as a ratio, this means for every 3 units of alcohol, there is 7 units of water, because 3/10 = 30%.

Answer:

3 : 7

Step-by-step explanation:

We can do this question using a easier way.

For example, A glass contains sugar and water, the ratio to sugar to water is 2 : 5. And the second glass's sugar to water is 2 : 3. The mixture of the first glass is 200 mL. The mixture of the second glass is the same as the first glass.

This kind of glass question, is very important to Math.

Try using this to solve this question.

A standard deck of cards contains 52 cards. Of these cards there are 13 of each type of suit (hearts, spades, clubs, diamonds) and 4 of each type of rank (A - K). Two cards are pulled in order from this deck of 52 playing cards. What is the probability that the cards will be two 10's?
A) 1/663
B) 9/26
C) 1/252
D) 1/221

Answers

The probability of drawing two 10's is P = 1/221, so the correct option is D.

How to find the probability?

We know that in the deck of 52 cards, we have 4 10's.

Then, the probability of drawing the first 10 is:

p = 4/52.

At this point, we have 3 10's in the deck, and a total of 51 cards (because we already took one 10).

The probability of getting another is:

q = 3/51

The joint probability (of getting both 10's, one after the other) is given by the product of the individual probabilities:

P = p*q = (4/52)*(3/51) = (1/13)*(1/17) = 1/221

Then the correct option is D.

If you want to learn more about probability:

https://brainly.com/question/25870256

#SPJ1

A math teacher randomly assigns seats in the classroom to students each quarter. Each student randomly chooses their row number out of a hat. In the classroom, there are 4 desks in each row and 27 students in your class. What is the probability that you and your 3 friends will get to sit together in the same row?

Answers

Using the combination formula, it is found that the probability that you and your 3 friends will get to sit together in the same row is given by:

[tex]p = \frac{1}{17550}[/tex].

We do not consider the order, hence the combination formula is used to solve this question.

What is the combination formula?

[tex]C_{n,x}[/tex] is the number of different combinations of x objects from a set of n elements, given by:

[tex]C_{n,x} = \frac{n!}{x!(n-x)!}[/tex]

4 students will be taken from a set of 27, hence the number of ways they can sit is given by:

[tex]C_{27,4} = \frac{27!}{4!23!} = 17550[/tex]

Only one outcome is desired, hence the probability is given by:

[tex]p = \frac{1}{17550}[/tex].

More can be learned about the combination formula at https://brainly.com/question/25821700

#SPJ1

A number of friends decided to go on a picnic and planned to spend rs. 96 on eatables. four of them, however, did not turn up. as a consequence, the remaining ones had to contribute rs. 4 each extra. the number of those who attended the picnic was

Answers

The number of those who attended the picnic was 8.

Quadratic equations are the polynomial equations of degree 2 in one variable of type f(x) = [tex]ax^{2} + bx + c[/tex] = 0 where a, b, c, ∈ R and a ≠ 0.

Such questions are best examples of quadratic equations where  the unknown is simply assumed to be a variable which when substituted according to the conditions yield a quadratic equation which can easily be solved.

Let x represent the whole population.

Total Spending = Rs. 96

Thus, each individual's contribution is equal to 96/x.

Four people did not show up causing

others to contribute Rs. 4 extra.

So the given condition can be written as

96/(x-4) – 96/x = 4

=>96x – 96(x-4) = 4x(x-4)

=> 96x -96x + 384 = 4x2 – 16x

=> 4x2 – 16x – 384 = 0

=> x2 – 4x – 96 = 0

=> (x – 12)(x + 8) = 0

=> x = 12 or x = -8 (neglect)

So x = 12

x-4 = 8  ( because  four of them did not turn up)

∴ The number of those who attended the picnic was 8.

Learn more about quadratic equations here :

https://brainly.com/question/15241362

#SPJ4

Of the cartons produced by a company, 5% have a puncture, 8% have a smashed corner, and 0.4% have both a puncture and a smashed corner. Find the probability that a randomly selected carton has a puncture or a smashed corner.

Answers

The probability that a randomly selected carton has a puncture or a smashed corner is 12.6%.

In this problem, the events are:

Event A: Puncture.

Event B: Smashed corner.

The "or" probability is given by:

[tex]P(AUB)=P(A)+P(B)-P(A[/tex]∩[tex]B)[/tex]

5% have a​ puncture, hence [tex]P(A)=0.05[/tex]

8% have a smashed​ corner, hence [tex]P(B)=0.08[/tex]

0.4% have both a puncture and a smashed corner, hence[tex]P(AUB)=0.004[/tex]

Then:

[tex]P(AUB)=0.05+0.08-0.004= 0.126[/tex]

Therefore, The probability that a randomly selected carton has a puncture or a smashed corner is 12.6%.

Learn more about probability here https://brainly.com/question/20344684

#SPJ4

Find the tangent of angle Θ in the triangle below.

Answers

Answer: 4/3

Step-by-step explanation:

[tex]\tan \theta=\frac{20}{15}=\boxed{\frac{4}{3}}[/tex]

The second highest point measured above sea level is the summit of
Kangchenjunga which is 8,586m above sea level and the lowest point is
challenger deep at the bottom of Mariana Trench which is 10,911 m below
the sea level. What is the vertical distance between these two points? Collect
some more information about these two points and present them withimages

Answers

Answer:

To find the vertical distace between both points you will have to add both. 8,586+10,911= 19 497.

Hence, the vertical distance is 19,497 feet.

Here is a picture:
Hope this helps and please mark as brainliest!!

From the diagram below, if the sides AD = 3 and DC = 27, and BD = X + 3, find x.





Select one:

a.
36


b.
9


c.
16


d.
6

Answers

Answer: d. 6.

Step-by-step explanation:

AD=3      DC=27       BD=x+3      x>0     x=?

1. Consider triangle ABD:

It's rectangular (∡ADB=90°).       ⇒

AB²=(x+3)²+3²=(x+3)²+9.

2. Consider triangle BDC:

It's rectangular (∡BDC=90°).       ⇒

BC²=(x+3)²+27²=(x+3)²+729.

3. Consider triangle ABC:

It's rectangular (∡ABC=90°).      

AC=AD+CD=3+27=30.      ⇒

AC²=AB²+BC²

(x+3)²+9+(x+3)²+729=30²

2*(x+3)²+738=900

2*(x²+6x+9)+738=900

2x²+12x+18+738=900

2x²+12x-144=0 |:2

x²+6x-72=0

D=324   √D=18

x₁=-12 ∉ 'cause x>0

x₂=6.

3 quick algebra 1 questions for 50 points!

Only answer if you know the answer, shout-out to subtomex0, tysm for the help!

Answers

The range of the function across the domain is {18, 2, -6}

The range of the function

The function is given as:

f(x) = -2x + 12

The domain is given as:

{-3, 5, 9}

So, we have:

f(-3) = -2*-3 + 12 = 18

f(5) = -2*5 + 12 = 2

f(9) = -2*9 + 12 = -6

Hence, the range of the function across the domain is {18, 2, -6}

The value of the function

The function is

f(x) = 3x - 11

To calculate f(4), we have:

f(4) = 3 * 4 - 11

Evaluate

f(4) = 1

Hence, the value of f(4) is 1

The value of the function

The function of the graph is given as:

w(x)

To calculate w(2), we check the corresponding point on the graph where x = 2

From the graph, we have:

w(2) = 5

Hence, the value of w(2) is 5

Read more about domain and range at:

https://brainly.com/question/1632425

#SPJ1

Answer:

1.  f(4) = 1

2.  range: {18, 2, -6}

3.  w(2) = 5

Step-by-step explanation:

Question 1

[tex]\textsf{Given}: \quad f(x)=3x-11[/tex]

To find f(4) substitute x = 4 into the given function:

[tex]\begin{aligned}\implies f(4) & = 3(4)-11\\& = 12-11\\& = 1\end{aligned}[/tex]

Question 2

Domain: set of all possible input values (x-values)

Range: set of all possible output values (y-values)

[tex]\textsf{Given}: \quad b(x)=-2x+12[/tex]

To find the range of the given function for the domain {-3, 5, 9} simply input these values of x into the function:

[tex]\implies b(-3)=-2(-3)+12=18[/tex]

[tex]\implies b(5)=-2(5)+12=2[/tex]

[tex]\implies b(9)=-2(9)+12=-6[/tex]

Therefore, the range of the given function is {18, 2, -6}.

Question 3

Given:  The graph of function w(x)

To find w(2) simply find the y-value of the point on the graph where x = 2:

Find x = 2 on the x-axis.  Trace vertically until you meet the line.  Trace horizontally to the y-axis to find the corresponding value of y.

Therefore, w(2) = 5

There were some people on a train
17 get off the train at the first stop and 22 people get on the train
Now there are 66 people on the train
How many people were on the train to begin with

Answers

i think the answer is 31
You minus 66 by 22 which is 44 than you add 17 which is 61 so the answer is 61

The height of an
equilateral triangle is
11√12. What is a side
of the triangle?

Answers

Answer:

I don't know the answer

Step-by-step explanation:

it's hard solve it for me

Find the value of x.

Answers

Angles sum up to:( n - 2 ) × 180 = ( 5 - 2 ) × 180 = 3 × 180 = 540

[tex]x + 4x + 4x + 135 + 135 = 540[/tex]

[tex]9x + 270 = 540[/tex]

[tex]9x = 540 - 270 \\ 9x = 270[/tex]

[tex]x = \frac{270}{9} = 30[/tex]

This table shows the scores that Mario had on his last five math tests.

Which statement does NOT describe this set of data?

A. The mode is 85.
B. The mean is greater than the median.
C. The median is 81.
D. The mode and the median are equal.

Answers

D. The mode and the median are equal.

I think that should be your answer

Answer:

Step-by-step explanation:

its A i just did it lol

In the following exercises, multiply the binomials. Use any method.
254. (10a − b)(3a − 4)

Answers

Answer:

Hence the expression  [tex]$$(10a-b)(3a-4)=30a^2-3ab-40a+4b$$[/tex]

Step-by-step explanation:

Explanation

The given expression is (10a-b)(3 a-4).We have to multiply the given expression.Multiply the (10a-b) by -4, multiply the (10 a-b) by 3a then add like terms.

[tex]$$\begin{matrix}{} & {} & {} & {} & 10a & - & b \\ \times & {} & {} & {} & 3a & - & 4 \\ \end{matrix}$$[/tex]

___________________

[tex]$$\begin{matrix}{} & {} & {} & - & 40a & + & 4b \\ 30{{a}^2} & - & 3ab & {} & {} & {} & {} \\ \end{matrix}$$[/tex]

___________________

[tex]$$\begin{matrix}30{{a}^2} & - & 3ab & - & 40a & + & 4b \\ \end{matrix}$$[/tex]

Can someone give me the answer to this?

Answers

[tex]\frac{FI}{IG}=\frac{14}{28}=\frac{1}{2}\\\\\frac{FE}{EH}=\frac{12}{26}=\frac{6}{13}[/tex]

So, IE is not parallel to GH.

What is the domain of function m based on the graph?

Answers

Answer:

let why is equal to FX b and function with an independent variable X and dependent variable y if a function of f provide a way to successfully produced the single value using for the prepos a value of x then the chosen X value is called belong to domain f

Can someone solve it step by step for me please?

Answers

Okay, this is a simplified version of how to solve this complicated equation.

First add like terms but in this equation is not very efficient when solving.

So subtract the -2x squared from both sides and the equation should like mine

x⁴ + 4x³ + 12x + 45 = x ² -2x ² + 6x + 9

So keep the left side of the equation the same, but change the right like this because your adding the x squared.

-x² + 6x + 9

Now go back to the whole equation

x⁴ + 4x³ + 12x + 45 = -x² + 6x + 9

You can’t do anything on the left side of the equation do the same process with the other numbers like this:

x⁴ + 4x³ + 12x = -x² + 6x + 9 -45

You subtract the 45 from each side and it’s negative on the other side because you subtracted it.

Now add the 9 and -45 = -36

Now the equation should look like this:

x⁴ + 4x³ + 12x = -x² + 6x -36

Subtract -12x from each side and the 12x on the left side of the equation is canceled out and is gone

The equation should look like this:

x⁴ + 4x³ + = -x² + 6x -36 -12x

Then add the 6x and -12x because they are like terms

So this is the final simplified version of the complex problem

x⁴ + 4x³ + = -x² + -36 -6x

You can’t add any other like terms to find the value of x because x and x² can’t be added together

The perimeter of the rectangle below is 130 units. Find the length of side VY.
Write your answer without variables.
Y
4y
V
5y + 2
X
W
VY

Answers

Answer:

VY = 28 units

Step-by-step explanation:

the perimeter (P) of a rectangle is calculated as

P = 2 ( length + breadth )

given P = 130 , then

2(5y + 2 + 4y) = 130 ( divide both sides by 2 )

9y + 2 = 65 ( subtract 2 from both sides )

9y = 63 ( divide both sides by 9 )

y = 7

Then

VY = 4y = 4 × 7 = 28

(bus, cab, or train) to school, and in the evening he has the same 333 choices for his trip home.

Answers

The probability that he will use both bus and the train among bus, cab, train is 0.11.

Given There are 3 choices to people each among bus, cab, train for his trip home.

Probability is the likeliness of happening an event among all the events possible. The value of probability lies between 0 and 1. The formula to calculate probability is as under:

Probability= number of things/ total things.

The value of probability cannot be negative.

Number of buses=3

Number of cab=3

Number of train=3

Probability that he will go through train and bus is 3/9*3/9

=9/81

=1/9

=0.11

Hence the probability that he will travel with bus and train is 0.11.

Learn more about probability at https://brainly.com/question/24756209

#SPJ4

Question is incomplete as it includes:

What is the probability that he will use both train and bus?

17-19, deal with the ambiguous SSA. For each, find all possible solutions and sketch the triangle(s) in each case

Answers

The possible solutions using the Sine Law are given as follows:

Triangle A:
B = 50.26°

C = 79.74°

Triangle B:

B = 62.79°

C = 70.21°

What is the sine Rule?

The sine law asserts that the ratio of a triangle's side length to the sine of the opposing angle is the same for all three sides. The sine rule is another name for it.

What is the explanation for the above solution?

Triangle A

Given the sine law:
a/SinA = b/SinB

Hence

Sin B = (b/a) SinA

Sin B = 15/10 (Sin 50)

Sn B = (15/10) * 0.766

Sin B = 1.5 * 0.766

B = Sin⁻1 (1.14)

B = 50.26°

Hence, C =

180 - B - A

= 180 - 50.26 - 50

C = 79.74°

Solution B

Given the sine law:

a/SinA = b/SinB

Hence

Sin B = (b/a) SinA

Sin B = 2/1.6 (Sin 47)

Sn B = 1.25 * 0.73

Sin B = 0.9125

B = Sin⁻1 (0.9125)

B = 62.79°

Hence,

C = 180 - 62.79 - 47

C = 70.21°

Learn more about the Sine Law at;
https://brainly.com/question/12271113
#SPJ1

If two triangles are congruent which of the following statements must be true

Answers

The statement which must be true are Options B, C and D

which are the triangles have the same size and shape, the corresponding sides of the triangles are congruent and the corresponding angles of the triangles are congruent.

What are congruent triangles?

Congruent triangles are triangles that have corresponding sides, angles which are all equal in measure.

They can be rotated and turned to be look be identical.

Thus, the statement which must be true are Options B, C and D

which are the triangles have the same size and shape, the corresponding sides of the triangles are congruent and the corresponding angles of the triangles are congruent.

The complete question is

If two triangles are congruent which of the following statements must be true? CHECK ALL THAT APPLY

A. The triangles have the same size but not the same shape.

B. The triangles have the same size and shape

C. The corresponding sides of the triangles are congruent.

D. The corresponding angles of the triangles are congruent.

Learn more about congruent triangles here:

https://brainly.com/question/1675117

#SPJ1

Keilantra has a toy car collection of 400 toy cars. She keeps 268 of the toy cars on her
wall. What percentage of Keilantra's toy car collection does she keep on her wall?
Answer:
15
I
Submit Answer
MacBook Air
33
attempt 1 out of 2
4

Answers

Answer:

67%

Step-by-step explanation:

268 ÷ 400 = 0.67 = 67%

[tex]\huge\boxed{67\%}[/tex]

This can be solved just with division.

[tex]\dfrac{268\ \text{cars on the wall}}{400\ \text{total cars}}=0.67[/tex]

Multiply the result by [tex]100[/tex] to get a percentage.

[tex]0.67=\boxed{67\%}[/tex]

Suppose the round-trip airfare between Philadelphia and Los Angeles a month before the departure date follows the normal probability distribution with a mean of $387.20 and a standard deviation of $68.50. The interval around the mean that contains 95% of the airfares is ________.

Answers

The interval around the mean that contains 95% of the airfare is (250.2,524.2).

Normal Distribution is bell shaped and symmetrical in nature.

We use standard normal to find probabilities of normal distribution

Normal distributions have key characteristics that are easy to spot in graphs: The mean, median and mode are exactly the same. The distribution is symmetric about the mean—half the values fall below the mean and half above the mean. Diagram below shows standard normal figure.

In a normal distribution, 95% of the values fall between mean±2*sd

Give:  mean=387.20

          sd=68.50

So the interval around the mean that contains 95% of the airfare is (250.2,524.2).

To know more about Normal distribution visit:

https://brainly.com/question/4079902

#SPJ4


Select each row where the property is being used correctly.
-3x - 4x + 4
2(x - 5) + 1 ≤ 5
-4x ≥-24
x = 2y and 2x + 2y > 60
y+ 2 ≤4-x and 4 - x ≤ 3y


-3x < x
2(x - 5) ≤ 4
x≤6
X> 27
6y > 60
y + 2 ≤ 3y

Answers

The correct rows in the inequalities are

2(x - 5) + 1 ≤ 5  ⇒ 2(x - 5) ≤ 4-4x ≥ -24  ⇒ x ≤ 6x = 2y and 2x + 2y > 60  ⇒ 6y > 60y + 2 ≤ 4 - x and 4 - x ≤ 3y  ⇒ y + 2 ≤ 3y

How to determine the correct rows?

The rows are given as:

-3x - 4 < x + 4  ⇒ -3x < x

2(x - 5) + 1 ≤ 5  ⇒ 2(x - 5) ≤ 4

-4x ≥ -24  ⇒ x ≤ 6

x = 2y and 2x + 2y > 60  ⇒ 6y > 60

y + 2 ≤ 4 - x and 4 - x ≤ 3y  ⇒ y + 2 ≤ 3y

To determine the correct rows, we simply solve each inequality.

This is done as follows:

-3x - 4 < x + 4

Collect like terms

-3x - x < 4 + 4

Evaluate the like terms

-4x < 8

Divide by -4

x > -2

This means that:

-3x - 4 < x + 4  ⇒ -3x < x is false.

2(x - 5) + 1 ≤ 5  

Subtract 1 from both sides

2(x - 5) ≤ 4

This means that:

2(x - 5) + 1 ≤ 5   ⇒ 2(x - 5) ≤ 4 is true.

-4x ≥ -24  

Divide through by -4

x ≤ 6

This means that:

-4x ≥ -24  ⇒ x ≤ 6 is true.

x = 2y and 2x + 2y > 60

Substitute 2y or x in 2x + 2y > 60

2(2y) + 2y > 60

Evaluate the product

4y + 2y > 60

Evaluate the like terms

6y > 60

This means that:

x = 2y and 2x + 2y > 60  ⇒ 6y > 60 is true.

y + 2 ≤ 4 - x and 4 - x ≤ 3y

We have:

y + 2 ≤ 4 - x and 4 - x ≤ 3y

This can be rewritten as:

y + 2 ≤ 3y

This means that:

y + 2 ≤ 4 - x and 4 - x ≤ 3y  ⇒ y + 2 ≤ 3y is true

Hence, the correct rows are 2, 3, 4 and 5

Read more about inequality at:

https://brainly.com/question/17675534

#SPJ1

As strange as it may seem, it is possible to give a precise-looking verbal definition of an integer that, in fact, is not a definition at all. The following was devised by an English librarian, G. G. Berry, and reported by Bertrand Russell. Explain how it leads to a contradiction. Let [tex]n[/tex] be “the smallest integer not describable in fewer than [tex]12[/tex] English words.” (Note that the total number of strings consisting of [tex]11[/tex] or fewer English words is finite.)

Worth 10 points.
YOUR ANSWER MUST BE UNIQUE

Answers

The statements hold the contradiction, just by confusion in keep meaning of statements and get confused with "12" which is string in quotes

Let  be “the smallest integer not describable in fewer than  English words.” (Note that the total number of strings consisting of  or fewer English words is finite.)

What do you mean by contradiction?
Contradiction means that couple of statements that opposes each other.

Here,
The statement says let n be  “the smallest integer not describable in fewer than  English words.” which implies that n is connected with string mentioned in "" quotes, which is not a definition but contain 11 words in the quotes.
and another statement -  Note that the total number of strings consisting 11  or fewer English words is finite which implies that:
the limit of words inside the quotes 11 words

Thus, strings implies in quotes is simple meaning with 11 word in it
While in statement in brackets implies the limit of words as a string should be ≤ 11. the string "12" create the contradiction in minds.

Learn more about contradiction here:
https://brainly.com/question/13711793

#SPJ1

Other Questions
What is the probability of obtaining in a row when flipping a coin?. How was America's reliance onEnglish manufactured productsimpacted by the IndustrialRevolution?A. America's reliance on products from Englandincreased.B. America's reliance on products from Englanddecreased.C. America's reliance on products from Englandremained the same.D. America's reliance on products from Englandended. Is there any difference(s) between soil profile and soil structure? If so kindly explain. What is an application of a magnetic effect of an electric current?And how does the application work due to the magnetic effect of an electric current? please help i dont get it Simplify the following expression. Which word most affects the tone of this sentence?A slender woman walked into the room wearing a pink dress and a gaudy hat.O gaudyO pinkO slenderO woman Read the excerpt from Ovid's "Pyramus and Thisbe".Which statement best describes how the order ofPyramus had leftevents creates tension?a little later than his Thisbe had. Because Pyramus does not see the beast, readersand he could see what surely were the tracksfear confrontation.of a wild beast left clearly on deep dust. Because Pyramus does not see the beast, heHis face grew ashen. And when he had foundbelieves he is lost.the bloodstained shawl, he cried: "Now this same nightwill see two lovers lose their lives: she wasO Because Pyramus is late, he misunderstandsthe one more worthy of long life: it'sThisbe's situation.who bear the guilt for this. Because Pyramus is late, readers question hisdevotion. Which pair of expressions is equivalent using the Associative Property of Multiplication? 4(2a 5) = (4 2a) 5 4(2a 5) = 8a 20 4(2a 5) = (2a 5) 4 4(2a 5) = 4 2a 5 You are a research scientist studying bioluminescent bacteria (bacteria that can glow). You grow several plates of bacteria and give them a special chemical that you hypothesize will increase their luminescence, or brightness. You turn off the lights and use a device called a photometer to measure the light levels (in lumens) coming from each plate. You collect the following data:Plate 1: 7 lumens, 6 lumens, 13 lumensPlate 2: 4 lumens, 5 lumens, 7 lumensPlate 3: 3 lumens, 6 lumens, 7 lumensPlate 4: 5 lumens, 5 lumens, 8 lumensWhich of the following pieces of data is likely an outlier? The B737-400 aircraft costs $2948 dollars per hour to operate. The algebraic expression 2948t gives the total cost to operate the aircraft for t hours. Find the total cost to operate the B737-400 for 3.6 hours. Make up a python program that can do the followingWrite a program to ask the user to input the number of hours a person has worked in a week and the pay rate per hour. Compare these three ratios using fractions.3 to 2 5:6 8 to 12Think about the steps you could take to compare the ratios using fractions.The first step is to write the ratios as fractions.The next step is to use a to rewrite the fractions.Finally, compare the to help you order the ratios. In each case below, a charged rod, made of the dense rubber ebonite, comes close or is in contact with the top of an electroscope. The ball on top of the electroscope is directly connected to the two metal leaves suspended in the flask.Which image represents a gaining of a charge on the leaves of the electroscope by conduction? Please help!!! In 2000, the median household income was $41,990. In 2010, it was $49,445. What is the average rate of change in the median household income over this time period? Multiply (-3) by (+1,4 Mr Ade's monthly salary is #460 after paying #50 for his house, giving #2 to each 5 children and setting aside #290 for food, how much has he left? What is the multiplicative rate of change of the function?0.20.250.50.75 What is the simplified form of 4 log3y - 6 log3x + 7 log3z? What has to be true for the triangles to be congruent?